Please help me answer this question please

Please Help Me Answer This Question Please

Answers

Answer 1

Answer:

B

Step-by-step explanation:

ok so let us start with the bases its 4*3 which is 12 and there are 2 so 12+12 and the bottom is 4*10 so its 12+12+40 then the back is 3*10 so its 12+12+40+30 and the front is easy 5*10 so 12+12+40+30+50 is the correct answer

Make me brainliest please


Related Questions

Interest = $821.25, time = 1 year, rate = 12.42%. Find the principal

Answers

Step-by-step explanation:

P = l.100/t.r = $821.25×100/12.42×1 = $6612.318

hope this helps you.

You are a salesman for company Z. Total sales for company Z last year were
R900 000,00. You were responsible for obtaining R300 000,00 worth of these
sales. As a fraction, what was your share of the total sales?

Answers

Answer:

1/3 or 33.33%

Step-by-step explanation:

30000000/90000000

How many liters of 70% alcohol solution and 30% alcohol solution must be mixed to obtain 8 liters of 60% alcohol solution?

a. Complete the table below with the expressions needed to solve this problem. Then solve it in part b.

70% solution 30% solution Final solution
Number of Liters x y
Liters of Alcohol


b. You need
liters of 70% solution and
liters of 30% solution.

Answers

A Hi complete the table shown with an expression need to solve the problem and solve in

The figure below shows the graph of f’ the derivative of the function f, on the closed interval from x = -2 to x = 6. The graph of the derivative has horizontal tangent lines at x = 2 and x = 4.

find the x-value where f attains it’s absolute maximum value on the closed interval x=-2 to x=6. justify your answer

Answers

Answer:

  x = -2

Step-by-step explanation:

The function is decreasing where the first derivative is negative.

That is, the function is decreasing on the interval (-2, 5), except at x=2, where there is a flat spot. That means the maximum value is found at the left end of that interval, at x=-2.

__

The maximum value might be found at x = 6, at the right end of the interval (5, 6) on which the function is increasing. However, we judge the area under the first derivative curve between x=5 and x=6 to be less than the total area under the curve between x=-2 and x=5. That means the function does not increase enough from its minimum at x=5 to make up for the decrease over the longer interval.

__

The attachment shows an approximation of the derivative function given in the problem statement (dashed line) and its integral (solid line). Not all of the curvature of f'(x) is accounted for in the approximation, but the overall result is consistent with the above analysis.

Determine the measure of the missing angle.
1 point
65°
?
57°
57 degrees
O 48 degrees
65 degrees
O 58 degrees
Solve for

Answers

Step-by-step explanation:

65° + 57° + x = 180° ( the sum of angle of triangle is 180° )

122° + x = 180°

x = 180° - 122°

x = 58°

hence the answer is 58°....

Answer:

58 degrees

Step-by-step explanation:

Sum of angles in a triangle is 180 degrees

180 = 65 + 57 + x

180 = 122 +x

x =180 -122

x = 58

Dont answer unless your sure will mark brainliest

Answers

Answer:

199.5

Step-by-step explanation:

ade and ola share a sum of 400 naira in the ratio 3:5. find the amount ade received.​

Answers

Answer:

150 Naira

Step-by-step explanation:

3 + 5 = 8

Since Ade receives in a 3 to 5 ratio, Ade receives 3/8 of the total.

3/8 × 400 = 150

Answer: 150 Naira

Some of the most important nutrients in the ocean include:
A. Nitrates, carbonic acid, and oxygen.
B. Carbon dioxide, oil, and gold.
C. Phosphate, phytoplankton, and potassium.
D. Phosphates, nitrates, and carbon.

Answers

Answer:

D one is the answer

Step-by-step explanation:

Write the tangent, cosine and sine ratios of angles X & Y. Write each answer as a (reduced) fraction. Not a decimal.

Answers

The tangent, cosine and sine ratios of angles X & Y in the reduced faction form is 3/4, 4/5 and 3/5 respectively.

What are the trigonometry ratios?

For a right angle triangle, the trigonometry ratios can be given as,

[tex]\rm \sin \theta=\dfrac{b}{c}\\\rm \cos \theta=\dfrac{a}{c}\\\rm \tan \theta=\dfrac{b}{a}[/tex]

Here, a is base side, b is perpendicular side and c is the hypotenuse side of the triangle.

In the given triangle, the length of base side is 8 units, perpendicular side is 6 units and hypotenuse side is 10 units.

[tex]a=8\\b=6\\c=10[/tex]

Thus, the  tangent, cosine and sine ratios of angles X & Y are,

[tex]\rm \sin \theta=\dfrac{6}{10}=\dfrac{3}{5}\\\rm \cos \theta=\dfrac{8}{10}=\dfrac{4}{5}\\\rm \tan \theta=\dfrac{6}{8}=\dfrac{3}{4}[/tex]

Thus, the tangent, cosine and sine ratios of angles X & Y in the reduced faction form is 3/4, 4/5 and 3/5 respectively.

Learn more about the trigonometry angles here;

https://brainly.com/question/20519838

#SPJ1

HELP PLS A rectangle has vertices at these coordinates.

(0, 8), (5, 8), (5, 0)

What are the coordinates of the fourth vertex of the rectangle?



Enter the coordinates in the boxes

Answers

Answer:

(0, 0).

Step-by-step explanation:

The 2 points with y = 8 is a horizontal side  with (5, 8) 5 units  to the right and  the point (5, 0) lies vertically down from (5, 8) so the fourth  point is 5 to the left of (5, 0) which is (0, 0)

Gretta is 2 1/2 kilometers tall. Which of the followings is equivalent to 2 1/2 kilometers in meters

Answers

Answer:

Gretta would be 2500 meters tall

Step-by-step explanation:

2km=2000 and the 1/2 would be 1/2=500 so the answer it 2500

Pls mark brainliest!!!!

Which expression is the result of solving the equation ar – b = cy for ? (For a 70).
cy
cy + b
+ b
Don

Answers

Answer:

4th option, (cy+b)/a

Step-by-step explanation:

By algebraic manipulation, we add b to both sides, then divide by a to isolate x.

Find the volume of the solid.

Answers

90 cu in.
V= L * W * H
Split shape into two figures

Alex buys 30 shares of Walmart at the close price if $48.80. His broker charges him 3% of each share. How much did Alex spend in total to buy this stock?

Answers

Alex spend $1507.92 in total to buy this stock at 3% charges on each shares.

What is shares?

Shares represent equity ownership in a corporation or financial asset, owned by investors who exchange capital in return for these units.

Alex buys 30 shares of Walmart at the close price of $48.80.

His broker charges him 3% of each share.

Purchasing of 30 shares =48.80 *30

                                         =$1464

Now we have to find the 3% of total purchasing of shares.

So,

Broker's fees =[tex]\frac{3}{100}*1464[/tex]

                      = 0.03 * 1464

                      = 43.92

So, total cost of stock= 1464+43.92

                                   = 1507.92

Hence, Alex spend $1507.92 in total to buy this stock.

Learn more about shares here:

https://brainly.com/question/2120956

#SPJ1

Segment AB has coordinates A(-4, 7) and B(5, 1). Find AP/AB and the coordinates of P that partitions AB such that AP:PB = 1:2. AP/AB= [Select] Coordinates of P: ( [Select] [Select] Hint: (x₁+ m/n (x2-x1). Y₁+ m/n (y2-V₁))​

Answers

[tex]\textit{internal division of a line segment using ratios} \\\\\\ A(-4,7)\qquad B(5,1)\qquad \qquad \stackrel{\textit{ratio from A to B}}{1:2} \\\\\\ \cfrac{A\underline{P}}{\underline{P} B} = \cfrac{1}{2}\implies \cfrac{A}{B} = \cfrac{1}{2}\implies 2A=1B\implies 2(-4,7)=1(5,1)[/tex]

[tex](\stackrel{x}{-8}~~,~~ \stackrel{y}{14})=(\stackrel{x}{5}~~,~~ \stackrel{y}{1})\implies P=\underset{\textit{sum of the ratios}}{\left( \cfrac{\stackrel{\textit{sum of x's}}{-8 +5}}{1+2}~~,~~\cfrac{\stackrel{\textit{sum of y's}}{14 +1}}{1+2} \right)} \\\\\\ P=\left(\cfrac{-3}{3}~~,~~\cfrac{15}{3} \right)\implies P=(-1~~,~~5)[/tex]

now, the segment AB cut by P in a 1:2 ratio, makes 3 thirds, so the ratio AP/AB is

[tex]\cfrac{AP}{AB}\implies \cfrac{1}{1+2}\implies \cfrac{1}{3}[/tex]

Patrick and Brooklyn are making decisions about their bank accounts. Patrick wants to deposit $300 as a principal amount, with an interest of 3% compounded quarterly. Brooklyn wants to deposit $300 as the principal amount, with an interest of 5% compounded monthly. Explain which method results in more money after 2 years. Show all work.

Answers

Answer:

Brooklyn's method results in more money after 2 years

Step-by-step explanation:

Compound Interest Formula

[tex]\large \text{$ \sf A=P(1+\frac{r}{n})^{nt} $}[/tex]

where:

A = final amountP = principal amountr = interest rate (in decimal form)n = number of times interest applied per time periodt = number of time periods elapsed

Patrick's Method

Given:

P = $300r = 3% = 0.03n = 4t = 2

Substituting the given values into the formula and solving for A:

[tex]\implies \sf A=300\left(1+\dfrac{0.03}{4}\right)^{4 \times 2}[/tex]

[tex]\implies \sf A=300\left(1.0075\right)^{8}[/tex]

[tex]\implies \sf A=318.4796543...[/tex]

Therefore, using Patrick's method, there would be $318.48 in the account after 2 years.

Brooklyn's Method

Given:

P = $300r = 5% = 0.05n = 12t = 2

Substituting the given values into the formula and solving for A:

[tex]\implies \sf A=300\left(1+\dfrac{0.05}{12}\right)^{12 \times 2}[/tex]

[tex]\implies \sf A=331.4824007...[/tex]

Therefore, using Brooklyn's method, there would be $331.48 in the account after 2 years.

As $331.48 > $318.48 then Brooklyn's method results in more money after 2 years.

Let X1, Xn be i.i.d. random variables with probability density function,

fx (x) = { 1/0 0 0 < x < 0 otherwise.

(a) Let X₁, X denote a bootstrap sample and let X = n

Find: E(XX1, , Xn), V(XX1, , Xn), E(X), V(X₂).

Hint: Law of total expectation: E(X)= E(E(XY)).

Law of total variance: V(X) = E(V(XY))+ (E(X|Y)).

Sample variance, i.e. s² = (X; - X)2 is an unbiased estimator of population variance.

Answers

The expected value of the assumed probability density function is 3/4 and the variance is 3/80

How to solve for E(x) and V(x)?

The probability density function is not properly stated.

So, I will provide a general explanation on how to determine E(x) and V(x) of a probability density function

Assume that the probability density function is given as:

[tex]f(x)= \left[\begin{array}{ccc}3x^2 &0 \le x \le 1\\0&Otherwise\end{array}\right[/tex]

The expression E(x) is calculated using:

[tex]E(x) = \int\limits^a_b {x \cdot f(x)} \, dx[/tex]

So, we have:

[tex]E(x) = \int\limits^1_0 {x \cdot 3x^2 \ d(x)[/tex]

Evaluate the product

[tex]E(x) = \int\limits^1_0 { 3x^3 \ d(x)[/tex]

Integrate

[tex]E(x) = \frac34x^4 |\limits^1_0[/tex]

Expand

[tex]E(x) = \frac34(1-0)^4[/tex]

Evaluate

[tex]E(x) = \frac34[/tex]

The variance is then calculated using:

[tex]Var(x) = E(x^2) - (E(x))^2[/tex]

Calculate [tex]E(x^2)[/tex] using:

[tex]E(x^2) = \int\limits^a_b {x^2 \cdot f(x)} \, dx[/tex]

This gives

[tex]E(x^2) = \int\limits^1_0 {x^2 \cdot 3x^2} \, dx[/tex]

Expand

[tex]E(x^2) = \int\limits^1_0 {3x^4} \, dx[/tex]

Integrate

[tex]E(x^2) = \frac{3}{5}x^5|\limits^1_0[/tex]

Expand

[tex]E(x^2) = \frac{3}{5}(1 - 0)^5[/tex]

Evaluate

[tex]E(x^2) = \frac{3}{5}[/tex]

Substitute [tex]E(x^2) = \frac{3}{5}[/tex] and [tex]E(x) = \frac34[/tex] in [tex]Var(x) = E(x^2) - (E(x))^2[/tex]

[tex]Var(x) = \frac 35 - (\frac{3}{4})^2[/tex]

Evaluate the exponent

[tex]Var(x) = \frac 35 - \frac{9}{16}[/tex]

Evaluate the difference

[tex]Var(x) = \frac{3}{80}[/tex]

Hence, the variance of the probability density function is 3/80

Read more about expected value and variance at:

https://brainly.com/question/15858152

#SPJ1

Identify the 17th term in the arithmetic sequence.
a1 = 11, d = −2

Answers

The arithmetic sequence has a common difference, and a first term and the 17th term in the arithmetic sequence is -21

How to determine the 17th term?

The given parameters are:

First term, a = 11Common difference, d = -2

The nth term of an arithmetic progression is:

Tn = a + (n - 1)d

So, we have:

T17 = a + 16d

Substitute the given values

T17 = 11 - 16 * 2

Evaluate

T17 = -21

Hence, the 17th term in the arithmetic sequence is -21

Read more about arithmetic sequence at:

https://brainly.com/question/6561461

#SPJ1

-4+2(3m+7)


Explain please

Answers

Answer:

6m + 10

Step-by-step explanation:

Distributive Property

a(b + c) = ab + ac

Solving

-4 + 2(3m + 7)-4 + 2(3m) + 2(7)-4 + 6m + 146m + 10

Which shows a true comparison? Select all that apply. A. 32. 03 > 32. 3 32.03>32.3 B. 3. 114 > 3. 112 3.114>3.112 C. 4. 003 < 3. 996 4.003<3.996 D. 0. 541 > 0. 145 0.541>0.145 E. 0. 141 < 0. 19

Answers

Answer:

B. 3.114 > 3.112

D. 0.541 > 0.145

E. 0.141 < 0.19

Step-by-step explanation:

In order to determine which decimal is greater or less than another, compare the place values from left to right. First, compare the whole numbers to the left of the decimal point. If they are not the same, the smaller decimal number is the one with the smaller whole number. This applies to the rest of the places.

----------------------------------------------------

32.3 has a greater tenths value than 32.03, so 32.03 < 32.3. Option A is false.

----------------------------------------------------

3.114 has a greater thousandths value than 3.112, so 3.114 > 3.112. Option B is true.

----------------------------------------------------

4.003 has a greater whole number than 3.996, so 4.003 > 3.996. Option C is false.

----------------------------------------------------

0.541 has a greater tenths value than 0.145, so 0.541 > 0.145. Option D is true.

----------------------------------------------------

0.19 has a greater hundredths value than 0.141, so 0.141 < 0.19. Option E is true.

hope this helps!

The inequalities which are truly compared are B, D and E.

Given certain inequalities.

We have to find the true comparisons.

We have to look at the corresponding digits of numbers which are compared from left to right. Greater numbers on the left will be the greater number.

A. 32. 03 > 32. 3

This is false since 32.3 is greater. In 32.03, 0 is in the tenth place, while 3 is in the tenth place for 32.3.

B. 3. 114 > 3. 112

This is true since in the thousandths place, 4 is greater than 2.

C. 4. 003 < 3. 996

This is clearly false, since in the whole part, 4 is greater than 3.

D. 0. 541 > 0. 145

This is true, since in the tenths place, 5 is greater than 1.

E. 0. 141 < 0. 19

This is also true, since in the hundredths place, 9 is greater than 4.

Hence the true comparisons are B, D and E.

Learn more about Inequalities here :

https://brainly.com/question/28823603

#SPJ2

John is making apple pies and apple cobblers to sell at the farmer’s market. A pie uses 4 cups of apples and 3 cups of flour. A cobbler uses 2 cups of apples and 3 cups of flour. When John sells the pies and cobblers at the farmer’s market, he will make $3.00 profit per pie and $2.00 profit per cobbler. Let x = the number of pies John makes. Let y = the number of cobblers John makes. Interpret the meaning of the vertex with the maximum profit in context of john’s baking.

Answers

Step-by-step explanation:

there are some pieces of information missing like the price of apples and flour. or the selling prices. or any restrictions like a fixed budget to buy ingredients or how many pies and cobblers he can make in a certain available time.

all we can say based on the given information :

one pie = 4a + 3f (a = apples, f = flour)

one cobbler = 2a + 3f

P(x, y) = 3x + 2y =

= x(selling price - 4a - 3f) + y(selling price - 2a - 3f)

the max. of P(x, y) is the maximum profit by selling pies and cobblers, which is without any further information +infinity.

the more pies and cobblers he sells, the bigger the profit - no limit.

In the following diagram, what is the value of x?
x=[x]​

Answers

Answer:

x=90°

Step-by-step explanation:

Too much work............. ....

dummy forgot his pin code. 20% of his pin code was 246.8 what was his pin code

Answers

Answer:

1234

Step-by-step explanation:

If 20% of his pin code was 246.8, we have to multiply 246.8 by 5 to get his pin code, which is 1234 (not safe btw.)

would really appreciate it if you hearted this answer

What is the distance between the y-intercept of the function f(x)=2(3)^x and the y-intercept of the function g(x) represented by the table below?

Answers

Answer:

A. 1

Step-by-step explanation:

y-intercept of f(x)

f(x) = 2(3)°f(x) = 2(0, 2)

y-intercept of g(x)

g(x) = (0, 3)

Distance

⇒ 3 - 2

A. 1

MARKING BRAINLEST IF CORRECT
Solve for x.
66
6
7
110

Answers

Answer:

x =6

Step-by-step explanation:

17x + 8 + 66 + 74 +110 = 360

17x + 258 =360

17x = 360 -258

17x =102

x =6

Apply angle sum property

17x+8+66+74+110=36017x+258=36017x=102x=6

HELP ASAP!!!

The black graph is the graph of y = f(x).
Choose the equation for the red graph.

Answers

Answer:

B

Step-by-step explanation:

We shift the black graph about the x axis, to do that we divide y by -1,

We then shift over to the left 5 units, to do that's we add 5 to the x value.

So the answer here is B

round 2.577681 to 4 decimal places

Answers

Answer:

2.5777

Step-by-step explanation:

Answer: 2.5777

Step-by-step explanation:

Given:

2.577681

Find the fourth decimal place:

2.577681

Round:

2.5777

Rounding:

Is the place to the right greater than or less than 5?

If greater than or equal to, round up. If less than, it stays the same.

    In this case, 8 > 5

The volume of a sphere is
[tex]125\pi[/tex] cm3. What is the radius?
Last question.​

Answers

Hi!!

Answer:

I'm not 100% sure, but I think it's 3.1.

Step-by-step explanation:

I took your volume, which is 125[tex]\pi[/tex], then a formula, which is

V=[tex]\frac{4}{3}[/tex][tex]\pi[/tex][tex]r^{3}[/tex], then I just plugged in the 125 and I got 3.1. I'm so sorry if it's wrong! <3

The image shows a circle centered at point O angle AOB Measures at 42 degrees.

What is the Measure of the angle acb explain your reasoning

Answers

The measure of angle ACB for circle centered at point O is 21°

What is an equation?

An equation is an expression that shows the relationship between two or more numbers and variables.

∠AOB = 2 * ∠ACB (angle at center is twice the angle at circumference)

42 = 2 * ∠ACB

∠ACB = 21°

The measure of angle ACB for circle centered at point O is 21°

Find out more on equation at: https://brainly.com/question/2972832

#SPJ1

Solve for r.

r= __ ?

Answers

r= \frac{q^{2} }{p^{3}} +1

Other Questions
In 1883 the Supreme Court declared the Civil Rights Act of 1875unconstitutional. What reason did they give for making this decision? What influence did Magdas interest in homemade crafts have on her art and the materials she uses?Use an example from the passage to support your answer. Whats the volume for this problem? Help me with this please giving brainlist In order to determine the distance between r=mx/546 you must travel throughout inner-subject quantities. Is this correct? Why? (Resistors in Series and Parallel) In which 120-V light bulb does the filament have greater resistance: a 60-W bulb or a 120-W bulb? If the two bulbs are connected to 120-V line in series, through which bulb will there be the greater voltage drop? What if they are connected in parallel? Explain your reasoning. In an archery competition, each archer shoots 3 arrows at a target. An archers score is calculated by measuring each arrows distance from the center of the target and finding the mean. One archer has a mean distance of 1.6 inches and a MAD of 1.3 inches in the first round. In the second round, this same archers arrows are farther from the center but more consistent. What values for the mean and MAD would fit this description for the second round? Find the equation of the line that passes through the points (12, 9) and (2,7) Cuntos aos tiene Claudia? can someone help me I pay you 50 in 1965, the population of country A was 2 714 000. in 2015 the population was 3 663 900. write the population in 2015 correct to 3 significant figures? In seals, the gene for the length of the whiskers has two alleles. The dominant allele (A) codes long whiskers and the recessive allele (a) codes for short whiskers. if one parent seal is pure long-whiskerd and the other is short-whiskered.what percent of offspring would have short whiskers? QUICK HELP ME?! In one paragraph, using your own words, choose one advertisement method promoting tobacco use and disprove it with facts. Be sure to include correct spelling and proper grammar. Joe has a cube shaped box that needs to be filled with packaging material If the length of one side 2 feet, then what is the volume of the box? Pretend that you are the President of the Philippines, write a speech stating your plans and programs about the following sectors:A. HealthB. EducationC. Peace and OrderD. EconomicE. Infrastructure A ball is thrown straight upward from ground level with a velocity of 18 m/s. How much time passes before t here ball strikes the ground? Marcia is using the rule Heart, Star, Star to make a repeating pattern. she wants the pattern to repeat 6 times. How many stars will be in Marcias pattern? A list should be introduced with a colon when the list is preceded bya strong opinion.a complete thought.a dependent clause.a descriptive phrase. Is garden a closed syllable word? Which of the following statements is TRUE? Only poor people file for bankruptcy. Only poor people file for bankruptcy. People who file for bankruptcy can never get credit again. People who file for bankruptcy can never get credit again. People who file bankruptcy are deadbeats who do not want to pay their bills. People who file bankruptcy are deadbeats who do not want to pay their bills. People who file bankruptcy pay higher interest because they are a higher risk to lenders.